If f(a)=g(a) and f(b)-g(b), prove they will have parallel tan lines

Click For Summary
To prove that two differentiable functions f and g have parallel tangent lines at some point in the interval [a, b], it is essential to apply the Mean Value Theorem (MVT). Given that f(a) = g(a) and f(b) = g(b), one can define h(x) = f(x) - g(x), which leads to h(a) = h(b) = 0. By applying Rolle's Theorem, it follows that there exists at least one point c in (a, b) where h'(c) = 0, indicating that f and g have parallel tangents at that point. The discussion further extends to show that if the conditions are relaxed to f(b) - f(a) = g(b) - g(a), the same conclusion holds true. Thus, the relationship between the functions ensures parallel tangent lines under both sets of conditions.
NWeid1
Messages
81
Reaction score
0
1. Homework Statement
(a) If f and g are differentiable functions on the interval [a,b] with f(a)=g(a) and f(b)=g(b), prove that at some point in the interval [a,b], f and g have parallel tangent lines.

(b) Prove that the result of part a holds if the assumptions f(a)=g(a) and f(b)=g(b) are relaxed to requiring f(b)-f(a)=g(b)-g(a).


2. Homework Equations



3. The Attempt at a Solution
I know to use the MVT, but besides that I'm lost.
 
Physics news on Phys.org
NWeid1 said:
1. Homework Statement
(a) If f and g are differentiable functions on the interval [a,b] with f(a)=g(a) and f(b)=g(b), prove that at some point in the interval [a,b], f and g have parallel tangent lines.

(b) Prove that the result of part a holds if the assumptions f(a)=g(a) and f(b)=g(b) are relaxed to requiring f(b)-f(a)=g(b)-g(a).


2. Homework Equations



3. The Attempt at a Solution
I know to use the MVT, but besides that I'm lost.

Let h(x) = f(x) - g(x).
From the assumptions in part a, h(a) = h(b) = 0. Now you can use Rolle's Theorem, a special case of the MVT.
 
I already got it, but thanks, though.
 
Question: A clock's minute hand has length 4 and its hour hand has length 3. What is the distance between the tips at the moment when it is increasing most rapidly?(Putnam Exam Question) Answer: Making assumption that both the hands moves at constant angular velocities, the answer is ## \sqrt{7} .## But don't you think this assumption is somewhat doubtful and wrong?

Similar threads

Replies
1
Views
2K
  • · Replies 2 ·
Replies
2
Views
2K
Replies
5
Views
2K
Replies
3
Views
1K
Replies
7
Views
2K
  • · Replies 9 ·
Replies
9
Views
2K
Replies
2
Views
1K
  • · Replies 30 ·
2
Replies
30
Views
4K
  • · Replies 19 ·
Replies
19
Views
2K
  • · Replies 9 ·
Replies
9
Views
2K